17

In a letter to Hardy, Ramanujan described a simple identity valid for $0<a<b+\frac 12$:

\begin{align} & \small\int\limits_{0}^{\infty}\frac {1+\dfrac {x^2}{(b+1)^2}}{1+\dfrac {x^2}{a^2}}\dfrac {1+\dfrac {x^2}{(b+2)^2}}{1+\dfrac {x^2}{(a+1)^2}}\dfrac {1+\dfrac {x^2}{(b+3)^2}}{1+\dfrac {x^2}{(a+2)^2}}\cdots \, dx \\[5mm] = &\ \dfrac {\sqrt{\pi}}2\dfrac {\Gamma\left(a+\frac 12\right)\Gamma\left(b+1\right)\Gamma(b-a+1)}{\Gamma(a)\Gamma\left(b+\frac 12\right)\Gamma\left(b-a+\frac 12\right)}\tag1 \end{align}

Which I find remarkable.

Questions:

  1. Has anyone discovered a way to prove $(1)$? If so, how do you prove it?
  2. Where did Ramanujan learn all of his integrational-calculus material (It doesn't appear in the Synopsis book)?
  3. Does anyone know a pdf or book where I can start learning advanced integration?

I'm wondering how you would prove $(1)$ and if there are similar identities that can be made. Wikipedia doesn't have any information.

Felix Marin
  • 89,464
Crescendo
  • 4,089
  • 3
    A 'simple identity'... – Leon Sot Jan 31 '17 at 06:55
  • 2
    See http://mathoverflow.net/questions/66812/ramanujans-eccentric-integral-formula – S.C.B. Jan 31 '17 at 08:06
  • 1
    I expect Ramanujan read some other math book(s) besides the Synopsis mentioned by Hardy. Likely at a rate of at least 2 lines per second. – DanielWainfleet Jan 31 '17 at 13:41
  • maybe one can show that the left hand and the right hand side have the same set of zeros and poles. this should fix the identity up to some constant which is hopefully not to difficult to obtain (this is more or less Liouvilles theorem if i remember correctly) – tired Jan 31 '17 at 18:08
  • 1
    Try the product formula for $\sin(x)$ and use some complex analysis / contour integrals – reuns Feb 01 '17 at 05:33
  • A PDF http://advancedintegrals.com/wp-content/uploads/2016/12/advanced-integration-techniques.pdf – Zaid Alyafeai Mar 01 '17 at 10:08

2 Answers2

14

Proposition 1 :$$\color{blue}{\displaystyle \sum\limits_{k=1}^\infty \dfrac{(-1)^{k-1}}{k}\;\zeta_H(k,a)x^k \; =\; \ln\left(\dfrac{\Gamma(a)}{\Gamma(a+x)}\right)\tag1}$$

Proof :

$$\begin{align*} & \displaystyle \sum\limits_{k=0}^{\infty}(-x)^{k}\zeta_H(k+1,a)\\ & \displaystyle = \sum\limits_{k,n=0}^{\infty} \dfrac{(-x)^k}{(n+a)^{k+1}}\\ & \displaystyle = \sum\limits_{n=0}^\infty \dfrac{1}{n+a}\sum\limits_{k=0}^\infty \left(\dfrac{-x}{n+a}\right)^k \\ & \displaystyle = \sum\limits_{n=0}^\infty \dfrac{1}{n+a+x} \\ & \displaystyle = -\psi(a+x)\end{align*}$$

Now integrating,

$$\displaystyle \sum\limits_{k=1}^{\infty} \dfrac{(-1)^{k-1}}{k}\zeta_H(k,a)x^k =\ln\left(\dfrac{\Gamma(a)}{\Gamma(a+x)}\right)\\ $$

Proposition 2 :$$\color{blue}{\displaystyle \prod\limits_{n=0}^{\infty} \left(1+\dfrac{x^2}{(n+a)^2}\right)\;=\; \dfrac{\Gamma^2 (a)}{\Gamma(a+ix)\Gamma(a-ix)}\tag 2} $$

Proof :

It is sufficient to evaluate the series,

$$\begin{align*} & \displaystyle\sum\limits_{n=0}^{\infty}\ln\left(1+\dfrac{x}{n+a}\right) \\ & = \displaystyle \sum\limits_{k=1}^\infty\sum\limits_{n=0}^\infty \dfrac{(-1)^{k-1}}{k}x^k \dfrac{1}{(n+a)^k} \\ & =\displaystyle \sum\limits_{k=1}^{\infty} \dfrac{(-1)^{k-1}}{k}\zeta_H(k,a)x^k \\ & =\displaystyle \ln\left(\dfrac{\Gamma(a)}{\Gamma(a+x)}\right)\end{align*}$$

$$\displaystyle \prod\limits_{n=0}^{\infty} \left(1+\dfrac{x}{n+a}\right) = \dfrac{\Gamma(a)}{\Gamma(a+x)}$$

Therefore,

$$\displaystyle \prod\limits_{n=0}^{\infty} \left(1+\dfrac{x^2}{(n+a)^2}\right)\;=\; \dfrac{\Gamma^2 (a)}{\Gamma(a+ix)\Gamma(a-ix)}$$

Proposition 3 : If $\; \displaystyle F(s)=\int\limits_0^\infty x^{s-1}f(x)\; dx\; $ then $$\color{blue}{\displaystyle \int\limits_{-\infty}^{\infty} |F(ix)|^2 \; dx \;= \; 2\pi\int\limits_0^\infty \dfrac{|f(x)|^2}{x}\; dx\tag 3} $$

Proof :

$$\displaystyle F(it)=\int\limits_0^\infty x^{it}\dfrac{f(x)}{x}\; dx$$

Set $x=e^y$ ,

$$\displaystyle F(it)=\int\limits_0^\infty e^{ixt}f(e^x)\; dx$$

Now by properties of Fourier Transform,

$$\begin{align*} & \displaystyle f(e^t)=\int\limits_{-\infty}^\infty g(x)e^{-ixt}\; dx \\ & \displaystyle g(t)=\dfrac{1}{2\pi}\int\limits_{-\infty}^\infty f(e^x)e^{ixt}\; dx\\\end{align*}$$

$$\begin{align*}\displaystyle F(it) & =2\pi g(t) \\ \displaystyle \int\limits_{-\infty}^\infty |F(it)|^2\; dt & = 4\pi^2 \int\limits_{-\infty}^\infty |g(t)|^2\; dt \\ \displaystyle \int\limits_{-\infty}^\infty |F(it)|^2\; dt & = 2\pi \int\limits_{-\infty}^\infty g(t) \int\limits_{-\infty}^\infty e^{ixt}f(e^x)\; dx\; dt \\ \displaystyle \int\limits_{-\infty}^\infty |F(it)|^2\; dt & = 2\pi \int\limits_{-\infty}^\infty f(e^x) \int\limits_{-\infty}^\infty e^{ixt}g(t)\; dt\; dx \\ \displaystyle \int\limits_{-\infty}^\infty |F(it)|^2\; dt & = 2\pi \int\limits_{-\infty}^\infty f(e^x)\overline{f(e^x)}\; dx =2\pi\int\limits_{-\infty}^\infty |f(e^x)|^2\; dx\end{align*}$$

Now by setting $e^x=t$ we get our result,

$$\displaystyle \int\limits_{-\infty}^\infty |F(it)|^2\; dt = 2\pi\int\limits_{-\infty}^\infty \dfrac{|f(t)|^2}{t}\; dt$$

Main Problem: $$\color{blue}{\displaystyle \int\limits_{0}^{\infty}\frac {1+\dfrac {x^2}{(b+1)^2}}{1+\dfrac {x^2}{a^2}}\dfrac {1+\dfrac {x^2}{(b+2)^2}}{1+\dfrac {x^2}{(a+1)^2}}\dfrac {1+\dfrac {x^2}{(b+3)^2}}{1+\dfrac {x^2}{(a+2)^2}}\cdots \, dx=\dfrac {\sqrt{\pi}}2\dfrac {\Gamma\left(a+\frac 12\right)\Gamma\left(b+1\right)\Gamma(b-a+1)}{\Gamma(a)\Gamma\left(b+\frac 12\right)\Gamma\left(b-a+\frac 12\right)}} $$

Proof : If we denote the integral by $I$ then using $(2)$ it can be rewritten as,

$$\displaystyle I = \dfrac{\Gamma^2 (b+1)}{\Gamma^2 (a)}\dfrac{1}{2} \int\limits_{-\infty}^\infty \dfrac{|\Gamma(a+ix)|^2}{|\Gamma(b+1+ix)|^2}\; dx$$

Now by defining $\displaystyle h(x)=\dfrac{x^a (1-x)^{b-a}}{\Gamma(b-a+1)}$ for $x\in[0,1]$ and $0$ for $\forall x\notin[0,1]$ we can conclude that $\displaystyle F(s)=M[h(x)]=\dfrac{\Gamma(s+a)}{\Gamma(s+b+1)}$ and from $(3)$ it follows that,

$$\displaystyle I = \dfrac{\Gamma^2 (b+1)}{\Gamma^2 (a)}\dfrac{1}{2} \int\limits_{0}^1 \dfrac{|h(x)|^2}{x}\; dx = \dfrac {\sqrt{\pi}}2\dfrac {\Gamma\left(a+\frac 12\right)\Gamma\left(b+1\right)\Gamma(b-a+1)}{\Gamma(a)\Gamma\left(b+\frac 12\right)\Gamma\left(b-a+\frac 12\right)}$$

where last line follows from the Duplication formula , and we are done !

$$\large \color{red}{\color{blue}{\boxed{\mathfrak{PROVED}}}} $$

1

\begin{align*} f(z) &= \prod_{\ell = 1}^{n} \left((b + \ell)^{2} + z^{2}\right), \quad g(z) = \prod_{\ell = 0}^{n} \left((a + \ell)^{2} + z^{2}\right) \\ \gamma_{R} &\text{ is the half-moon in the upper complex half-plane.} \\ f(i(a+k)) &= \prod_{\ell = 1}^{n} \left((b + \ell)^{2} - (a+k)^{2}\right) = \frac{(b+a+k+n)!}{(b+a+k)!} \cdot \frac{(b-a-k+n)!}{(b-a-k)!} \\ g'(i(a+k)) &= (-1)^{k} \cdot k! \cdot \frac{(2a+2k-1)!}{(2a+k-1)!} \cdot 2i(a+k) \cdot (n-k)! \cdot \frac{(2a+k+n)!}{(2a+2k)!} \\ &= i \cdot (-1)^{k} \cdot \frac{(2a+k+n)!}{(2a+k-1)!} \cdot k! \cdot (n-k)! \\ \int_{-\infty}^{\infty} &\frac{1}{a^{2}+x^{2}} \cdot \frac{(b+1)^{2}+x^{2}}{(a+1)^{2}+x^{2}} \cdots \frac{(b+n)^{2}+x^{2}}{(a+n)^{2}+x^{2}} \,dx \\ &= \lim_{R \to \infty} \oint_{\gamma_{R}} \frac{f(z)}{g(z)} \,dz = 2\pi i \sum_{k=0}^{n} \text{res}\left(\frac{f}{g}, i(a+k)\right) \\ &= 2\pi i \sum_{k=0}^{n} \frac{f\left(i(a+k)\right)}{g'\left(i(a+k)\right)} \\ &= 2\pi \sum_{k=0}^{n} (-1)^{k} \frac{(2a+k-1)!}{(2a+k+n)!} \frac{1}{k! (n-k)!} \frac{(b+a+k+n)! (b-a-k+n)!}{(b+a+k)! (b-a-k)!}. \end{align*}

If we multiply both sides by $a^{2}(a+1)^{2}\cdots(a+n)^{2} = \frac{(a+n)!^{2}}{(a-1)!^{2}}$ and divide by $(b+1)^{2}(b+2)^{2}\cdots(b+n)^{2} = \frac{(b+n)!^{2}}{b!^{2}}$, then \begin{align*} I_{n} &= 2\pi \frac{b!^{2}}{(a-1)!^{2}} \sum_{k=0}^{n} \underbrace{(-1)^{k} \frac{(2a+k-1)!}{k!} \frac{1}{(b+a+k)! (-2a-k)! (b-a-k)!}}_{A_{n,k}} \\ &\quad \times \underbrace{\frac{(b+a+k+n)! (b-a-k+n)!}{(2a+k+n)! (n-k)!} \frac{(a+n)!^{2}}{(b+n)!^{2}}}_{\text{denoted as } A_{n,k}}. \end{align*}

For $0 \leq k < M$, as $n \to \infty$, $\lim_{n \to \infty} A_{n,k} = 1$. Therefore, \begin{align*} I &= \lim_{n \to \infty} I_{n} \\ &= 2\pi \frac{b!^{2}(-2a)!(2a-1)!}{(a-1)!^{2}} \left(\sum_{k=0}^{M-1} \frac{A_{n,k}}{k! (b+a+k)! (-2a-k)! (b-a-k)!} + \sum_{k=M}^{n} \frac{A_{n,k}}{k! (b+a+k)! (-2a-k)! (b-a-k)!}\right) \\ &= 2\pi \frac{b!^{2}(-2a)!(2a-1)!}{(a-1)!^{2}} \left(\sum_{k=0}^{M-1} \frac{1}{k! (b+a+k)! (-2a-k)! (b-a-k)!} + \varepsilon_{M}\right), \end{align*} where $\varepsilon_{M}$ tends to zero as $M \to \infty$.

If $M \to \infty$, $\varepsilon_{M}$ tends to zero, and the hypergeometric series $\sum_{k=0}^{\infty} \frac{1}{k! (b+a+k)! (-2a-k)! (b-a-k)!}$ can be expressed as $\frac{(2b-2a)!}{(-2a)!(b-a)!(b-a)!(2b)!}$ for $2b-2a > -1$, or $a < b+\frac{1}{2}$. Therefore, [ I = 2\pi \frac{b!^{2}(2a-1)!(2b-2a)!}{(a-1)!^{2}(b-a)!^{2}(2b)!}. ]

This can be written using Legendre's duplication formula as \begin{align*} I &= \sqrt{\pi} \frac{\Gamma\left(a+\frac{1}{2}\right) \Gamma(b+1) \Gamma\left(b-a+\frac{1}{2}\right)}{\Gamma(a) \Gamma\left(b+\frac{1}{2}\right) \Gamma\left(b-a+1\right)}. \end{align*}

the integral (I) is expressed using Legendre's duplication formula as the square root of pi multiplied by the ratio of gamma functions: $$I = \sqrt{\pi} \frac{\Gamma\left(a+\frac{1}{2}\right) \Gamma(b+1) \Gamma\left(b-a+\frac{1}{2}\right)}{\Gamma(a) \Gamma\left(b+\frac{1}{2}\right) \Gamma\left(b-a+1\right)}$$

Martin.s
  • 1
  • 1
  • 22